Đến nội dung

manhtuan00 nội dung

Có 110 mục bởi manhtuan00 (Tìm giới hạn từ 08-06-2020)



Sắp theo                Sắp xếp  

#685688 Tuần 4 tháng 6/2017: Chứng minh rằng đường thẳng $AP$ luôn đi qua đ...

Đã gửi bởi manhtuan00 on 26-06-2017 - 22:30 trong Chuyên mục Mỗi tuần một bài toán Hình học

Lời giải bài 1 của em, hơi dài chút , đoạn sau có vẻ giống bạn dogsteven  ạ 

Gọi $U$ là điểm trên cung lớn $BC$ sao cho $AU \parallel BC$ . Bằng cộng góc ta có $\angle ABU = |\angle B - \angle C| = \angle ABL$ nên $BL,CK$ đi qua $U$ 

Gọi $R,S$ là tâm ngoại tiếp $\triangle AHM, \triangle AHN$. Ta có $\triangle OMC \sim \triangle ANH , \triangle ONB \sim \triangle ANH$ nên $\frac{KC}{LB} = \frac{R_{\triangle OMC}}{R_{\triangle ONB}} = \frac{R_{\triangle AMH}}{R_{\triangle ANH}} = \frac{AR}{AS} = \frac{AC}{AB}$ nên $\triangle AKC \sim \triangle ALB \implies AUKL$ nội tiếp

Gọi $W$ là giao 2 tiếp tuyến tại $B,C$ . $T$ là trung điểm $W$ , $Q$ là hình chiếu của $A$ lên $BC$ , $E$ là tâm Euler . Ta có $\angle AQE = \angle HAO = |\angle B - \angle C| = \angle ACK$ và $\frac{QE}{CK} = \frac{QE}{AR}. \frac{AR}{CK} = \frac{R}{2AR} . \frac{AH}{R} = \frac{AH}{2AR} = \frac{AQ}{AC} \implies \triangle AQE \sim \triangle ACK$ . Từ đây ta có $E$ là chân đường cao hạ từ $A$ xuống $KL$ . 

Gọi $\overline{X,Y,Z}$ là cát tuyến trực giao của $E$ với $\triangle ABC$ , $X,Y,Z$ lần lượt nằm trên $BC,CA,AB$ . Gọi $B',C'$ lần lượt đối xứng $B,C$ qua $E$ thì $B',C'$ là đối xứng của $O$ qua $CA,AB \implies Y,Z$ nằm trên trung trực $OC,OB$

$\implies Y \in TK , Z \in TL$ . Áp dụng định lý Desargues cho $\triangle ABC , \triangle TKL$ , giao điểm các cạnh là cát tuyến trực giao của $E$ với $\triangle ABC$ nên chúng thẳng hàng , suy ra $AP$ đi qua $T$ cố định

Untitled.png

Về phần sau thì em phát hiện ra 1 kết quả tổng quát hơn ạ

$\triangle ABC$ , đường thẳng qua $A$ song song $BC$ cắt $(O)$ tại $U$ . Một đường tròn bất kì đi qua $A,U$ cắt $UB,UC$ tại $K,L$ . Đường tròn $(K,KB) , (L,LC)$ cắt $(O)$ tại $X,Y$ . $BX \cap CY \equiv V , BL \cap CK \equiv W$ . Chứng minh rằng $A,W,V$ thẳng hàng

123.png

 

 

 




#685594 Tuần 4 tháng 6/2017: Chứng minh rằng đường thẳng $AP$ luôn đi qua đ...

Đã gửi bởi manhtuan00 on 26-06-2017 - 00:34 trong Chuyên mục Mỗi tuần một bài toán Hình học

Lời giải bài 2 của em ạ 

Gọi $R$ là giao điểm của $\Omega_1\Omega_2$ với $YZ$ . Gọi $\omega$ là góc Brocard . Ta có $\angle RZC = \angle RMC = \omega$ nên tứ giác $RZMC$ nội tiếp, từ đây có $\Omega_1Z.\Omega_1C = \Omega_1R.\Omega_1M  = \Omega_1A . \Omega_1X$. Tức là tứ giác $ARXM$ nội tiếp .  Gọi đường thẳng qua $C$ song song $\Omega_1\Omega_2$ cắt $NY$ tại $S$ . Khi đó $\angle YSC = \angle YN\Omega_2 = \angle YWC$ nên $S \in (O)$ . Do tứ giác $NSCM$ là hình thang cân nên $NS = CM$ .

Theo tính chất của điểm Brocard, ta có $O\Omega_1 = O\Omega_2 = R.\sqrt{\frac{\sum a^4}{\sum a^2b^2}-1}$ nên $S\Omega_2\Omega_1C$ cũng là hình thang cân

$\implies \triangle NS\Omega_2 = \triangle MC\Omega_1 \implies \angle S\Omega_2N = \angle C\Omega_1M = \angle N\Omega_1Z = \angle NYZ \implies $ tứ giác $NY\Omega_2R$ nội tiếp . Từ đây ta có $NS.NY = NR.N\Omega_2 = NA.NP$ nên tứ giác $PAR\Omega_2$ nội tiếp. Khi đó $\angle AP\Omega_2 = \angle ARN = AXQ$ , suy ra $P,Q, \Omega_2$ thẳng hàng

Untitled.png




#685415 Tuần 3 tháng 6/2017: Chứng minh rằng $\angle ADO= \angle OAG...

Đã gửi bởi manhtuan00 on 23-06-2017 - 19:29 trong Chuyên mục Mỗi tuần một bài toán Hình học

Lời giải bài 2 của em ạ : $\triangle APB \sim \triangle ACY$ nên $AP.AY = AB.AC$. Tương tự ta có $AQ.AX = AB.AC$ nên $\frac{AP}{AX} = \frac{AQ}{AY}$. Suy ra $PQ \parallel XY$. 

$PY \cap BC \equiv D_1 \implies \frac{D_1B}{D_1C} = \frac{PB}{PC}.\frac{YB}{YC}$. Vậy cần chứng minh $\frac{PB}{PC}.\frac{YB}{YC} = \frac{QC}{QC}.\frac{XB}{XC}$

$\leftrightarrow \frac{PB}{PC}.\frac{sin \angle AQB}{sin \angle AQC} = \frac{QB}{QC}.\frac{sin \angle APB}{sin \angle APC}$

$\leftrightarrow \frac{PB}{sin \angle APB} : \frac{PC}{sin \angle APC} = \frac{QB}{sin \angle AQB} : \frac{QC}{sin \angle AQC}$

$\leftrightarrow \frac{AC}{AB}.\frac{sin \angle ACP}{sin \angle ABP}. \frac{sin \angle PCB}{sin \angle PBC} = \frac{AC}{AB}.\frac{sin \angle ACQ}{sin \angle ABQ} . \frac{sin \angle QCB}{sin \angle QBC}$ ( điều này đúng do $\angle ACP = \angle QCB , \angle ABQ = \angle PBC, \angle PCB = \angle ACQ, \angle ABP = \angle QBC , \angle PBC = \angle ABQ$). Vậy $PY,QX,BC$ đồng quy tại $D$

Thật vậy, theo chứng minh trên thì $PQ \parallel XY$ nên $AD$ chia đôi $XY$ theo Menelaus , suy ra $AD$ chia đôi $PQ$




#685414 Tuần 3 tháng 6/2017: Chứng minh rằng $\angle ADO= \angle OAG...

Đã gửi bởi manhtuan00 on 23-06-2017 - 18:59 trong Chuyên mục Mỗi tuần một bài toán Hình học

Lời giải bài 1 của em ạ 

Điều cần chứng minh tương đương với $(DLJ)$ và $(O)$ trực giao. Sử dụng phép nghịch đảo $I^A_k$, ta đưa về bài toán : $\triangle ABC ; E,F \in CA,AB. H$ là trực tâm $\triangle AEF$. $(ABC) \cap AH, (AEF) \equiv D,M$. Khi đó tâm $(MDH)$ nằm trên $BC$

Gọi $EF$ cắt $BC$ tại $T$, $AH$ cắt $(AEF)$ tại $Q$

Gọi $U,V$ lần lượt là đối xứng của $M$ qua $BC,EF$. Khi đó $M,H,UV$ thẳng hàng trên đường thẳng Steiner của tứ giác toàn phần $(EC,CB,BF,FE)$ . Khi đó ta có $\angle MUH = 180 - \angle MTF = \angle MBF = \angle MDQ$ ( do $\triangle MDQ \sim \triangle MBF$ ). Từ đây ta có tứ giác $MDUH$ nội tiếp . Mà $BC$ là trung trực $MU$ nên tâm ngoại tiếp $(MDH)$ nằm trên $BC$ . Ta có điều cần chứng minh

Untitled.png




#685375 Đề thi IRAN TST 2017 - Phần 3

Đã gửi bởi manhtuan00 on 23-06-2017 - 09:59 trong Thi HSG Quốc gia và Quốc tế

Xin lỗi mình muốn hỏi tại sao

-Lại có điểm được đánh dấu 3 lần?

-Vì sao 3 đường tròn đó đi qua 5 điểm?

 

Tại sao

 

-$HGA'P$ là hình bình hành và $G$ nằm trên $OH$

-$\angle EGF = 360 - \angle EGB - \angle BGC - \angle CGF = 360 - (90 - \angle EBG / 2) - (90 - \angle FCG /2 ) - \angle BPC = \angle A +(\angle EBG + \angle FCG)/2 = \angle A + 90 - \angle GBP = 90 $ nên $\angle  EGF = 90 $ nên $\angle  EGF = 90$

Khúc này mình không hiểu lắm, với lại $EGF$ theo hình vẽ rõ ràng là góc tù sao $\angle  EGF = 90$

 

Cảm ơn bạn! 

em nghĩ thế này ạ 

Có $C^4_6 = 15$ tức là ta đã đánh dấu các điểm 15 lần > 2.6, theo nguyên lý Dirichlet thì có 1 điểm được đánh dấu 3 lần, tức là sẽ có 3 đường tròn sao cho phương tích từ điểm này đến 3 đường tròn đó đều bằng $k$ . Nếu chỉ cần một đường tròn đi qua điểm trên, tức là có 4 điểm đồng viên, thì ta có $k = 0$ thì sẽ suy ra được tất cả các điểm còn lại cũng nằm trên một đường tròn . Vậy xét trường hợp 3 đường tròn trên đều không đi qua điểm " được đánh dấu 3 lần " đó , tức là 3 đường tròn sẽ đi qua 5 điểm còn lại

Bài hình : ý đầu em làm hơi tắt tẹo, gọi $A'$ đối xứng $A$ qua $O$ . Tứ giác $BGCP$ là hình bình hành nên $G$ đối xứng $P$ qua trung điểm $M$, suy ra $HGA'P$ là hình bình hành , tức là $HG \parallel A'P$ . Mà $A'$ đối xứng $A$ qua $O$ , $P$ đối xứng $A$ qua $OH$ nên $HG \parallel OH$, kết hợp 2 điều trên ta có $G$ nằm trên $OH$

Ý thứ 2 : $\angel EGF = 90$ thì là theo phần cộng góc , và trong hình cũng hiển thị góc vuông ạ 

Hình gửi kèm

  • 19095668_1072510829547605_9200434862841217758_o.jpg



#684277 Tuần 2 tháng 6/2017: Chứng minh rằng $UV \perp AD$.

Đã gửi bởi manhtuan00 on 12-06-2017 - 22:04 trong Chuyên mục Mỗi tuần một bài toán Hình học

Một hướng tiếp cận khác cho bài 1 ạ 

Do $P$ đối xứng $D$ qua $KM$ nên $MP = MD = MA$ suy ra $M$ nằm trên trung trực $AP$, tương tự $N$ nằm trên trung trực $AQ$. Sử dụng phép vị tự tâm $A$ tỉ số 2, ta đưa về bài toán

$\triangle ABC, A'$ đối xứng $A$ qua $O$ , $(K)$ tiếp xúc $(O)$ tại $D$. $A'B,A'C$ cắt đường thẳng qua $A$ vuông góc $AD$ tại $F,E$. $EK,FK$ cắt $A'D$ tại $M,N$. Đường tròn $(AM),(AN)$ cắt $(K)$ tại $P,Q$. $NQ,MP$ lần lượt cắt $A'E, A'F$ tại $U,V$. Khi đó $UV \perp AD$. Gọi $R,S$ là 2 tiếp điểm của $(K)$ với $CA,AB$. Đường tròn $(AK)$ cắt $(O)$ tại $G$. Khi đó $K,G,O$ thằng hàng . 

Gọi $H$ là giao của tiếp tuyến tại $D$ với $BC$ , khi đó $GH$ cũng là tiếp tuyến của $(O)$ nên tứ giác $(GBDC)$ điều hòa . Từ đây ta có $A'(KD,EF) = (GD,BC) = -1$ nên $A'K$ chia đôi $EF$, suy ra $A'$ là trung điểm $MN$ do $EF \parallel MN$ 

Gọi $T$ là giao của $NQ$ và $MP$, do 3 trục đẳng phương của $(AT) , (K) , (O)$ đồng quy nên $PQ, DH, AG$ đồng quy , và trục đẳng phương của 3 đường tròn $(AK) , (K), (O)$ đồng quy nên 4 đường thẳng $AG,PQ,RS,DH$ đồng quy tại $W$ . 

Ta sẽ chứng minh  $G,T,A',K$ thẳng hàng , điều này tương đương với $T(\parallel MN , A',M,N) = A(DG,PQ)$. Thật vậy , ta có $T(\parallel MN , A',M,N) = -1$

Gọi $AD$ cắt $(K), PQ$ lần lượt tại $J,X$. Do tứ giác $JSDR$ điều hòa nên $WJ$ cũng là tiếp tuyến của $(K)$ . Vậy ta có $A(DG,PQ) = (WXPQ) = -1$. Vậy suy ra $T,A',K,G$ thẳng hàng 

Thật vậy, do $T,G,A'$ thẳng hàng thì theo phần trên ta có $A'(DT,VU) = -1$ nên $MV,NU , A'T$ đồng quy . Mà $A'$ lại là trung điểm $MN$ nên $UV \parallel MN$

Untitled.png




#683458 Đề thi IRAN TST 2017 - Phần 1

Đã gửi bởi manhtuan00 on 06-06-2017 - 23:40 trong Thi HSG Quốc gia và Quốc tế

Bài 5 : Gọi $M,N,R$ là trung điểm $BC,CA,AB$. $AM$ cắt $(APQ)$ tại $R$. Ta có $\angle MYQ = \angle QFC = \angle APQ = \angle MGQ$ nên $G \in (MQY)$, tương tự có $G \in (MPX)$. Khi đó $\angle GYT + \angle GXT = \angle GMP + \angle GMQ = 180 $ nên $G \in (TXY)$. Gọi $MN,MR$ cắt $AQ,AP$ tại $U,V$, bằng Menelaus ta sẽ có $UV \parallel BC$. Ta lại có $\angle MXG = \angle MPG = \angle MAU$ nên $MX.MU = MG.MA = MY.MV \implies UVYX$ nội tiếp 

Khi đó ta có $\angle GXY + \angle MAQ = \angle MXY - \angle MXG + \angle MAQ = \angle MVU - \angle MAQ + \angle MAQ = \angle C =\angle  QGY$ nên $(TXY)$ tiếp xúc $(APQ)$ tại $G$




#683455 Đề thi IRAN TST 2017 - Phần 3

Đã gửi bởi manhtuan00 on 06-06-2017 - 23:15 trong Thi HSG Quốc gia và Quốc tế

Bài 6 : Dựng hình bình hành $BGCP$ , $A'$ đối xứng $A$ qua $O$, $M$ là trung điểm $BC$. Khi đó $HGA'P$ là hình bình hành nên $G$ nằm trên $OH$

Ta có $\angle EGF = 360 - \angle EGB - \angle BGC - \angle CGF = 360 - (90 - \angle EBG / 2) - (90 - \angle FCG /2 ) - \angle BPC = \angle A +(\angle EBG + \angle FCG)/2 = \angle A + 90 - \angle GBP = 90 $ nên $\angle  EGF = 90 $ 

Gọi $Q$ là giao điểm của $(AEF)$ với $(O)$ . Khi đó ta có $\frac{QB}{QC} = \frac{EB}{FC} = \frac{PC}{PB}$ nên $PQ$ chia đôi $BC$ , tức là $Q,G,M,P$ thẳng hàng . GỌi $N$ là trung điểm $EF$

Xét phép vị tự quay tâm $Q$ biến $F \rightarrow C , N \rightarrow M$ nên $\triangle QMN \sim \triangle QCF$ nên $\angle QNM = \angle QCF = \angle QPA \implies MN \parallel AP \implies MN$ là trung trực $KG$ (do có $M$ là trung điểm $A'H$ ) , tức là $MK = MD$ nên $E,D,K,F$ đồng viên , suy ra $\angle EKF = 90 $




#683448 Đề thi IRAN TST 2017 - Phần 3

Đã gửi bởi manhtuan00 on 06-06-2017 - 22:14 trong Thi HSG Quốc gia và Quốc tế

Bài 4 : Xét bất kì 4 điểm , tồn tại 1 điểm có phương tích bằng $k$ đến đường tròn ngoại tiếp 3 điểm còn lại . Ta đánh dấu điểm này . Số cách chọn ra 4 điểm là $C^4_6 = 15$ nên tồn tại 1 điểm được đánh dấu 3 lần ( gọi là điểm $A$ ) , tức  là tồn tại 3 đường tròn mà phương tích từ điểm này chính là tâm đẳng phương của chúng. Nếu như có 4 điểm nào đó thuộc 1 đường tròn thì ta có $k=0$ thì tất cả mọi điểm nằm trên một đường tròn.Nếu 4 điểm bất kì không trên một đường tròn thì 3 đường tròn trên đi qua 5 điểm , suy ra sẽ có 2 đường tròn chung 2 điểm , 2 điểm này chính là trục đẳng phương (gọi là $X,Y$). Mà $A$ là tâm đẳng phương nên $A,X,Y$ thẳng hàng. Ta có điều mâu thuẫn




#682410 Tuần 5 tháng 5/2017: Chứng minh rằng bốn điểm $R,H,J,K$ cùng thuộc...

Đã gửi bởi manhtuan00 on 30-05-2017 - 13:14 trong Chuyên mục Mỗi tuần một bài toán Hình học

Lời giải bài 1 của em khá dài

Lời giải bài 1 :  Gọi $I$ là đối xứng của $A$ qua $O$. $S$ là giao của $HM$ với $EF$. $T$ là giao của $HM$ với $(O)$ . Theo một tính chất quen thuộc thì $SD$ đi qua giao 2 tiếp tuyến tại $B$ và $C$ ( kí hiệu là $V$ )  và $H,K,G$ thẳng hàng . Gọi $W$ là chân đường đối trung nằm trên $(O)$ . Khi đó $K,W$ đối xứng nhau qua $BC$ . Gọi $N$ là chân đường cao tại $A$ nằm trên $(O)$.Gọi $U$ là giao của $HQ$ với $BC$ thì $\angle PHQ  = \angle PLQ = \angle PDG$ . Suy ra $U \in (PHD)$ . Lại có $\angle SHD = 180^{\circ} - angle DLG = \angle DPG$ nên $S,H,D,P,U$ đồng viên. Gọi giao của $AO$ với $EF$ là $Y$

Ta cần chứng minh $R,H,J,K$ đồng viên, điều này tương đương với $\angle KJR = \angle GHU = \angle YPK$

Ta có $T(HA,XD) = -1$ mà $TH \perp TA$ nên $TH$ là phân giác trong $\angle DTK$. Từ đây có $\angle ITK = \angle DTH = \angle DGH$.  

Ta có $\angle SDT = \angle SDG - \angle GDT = \angle SYM - \angle TAK = \angle SYM - \angle THG = \angle SYM - \angle SYK = \angle MYK = \angle TIK$

Kết hợp 2 điều tển ta có $\triangle TSD \sim \triangle TKI \implies \angle TSD = \angle TKI = \angle HUG \implies \triangle HUG \sim \triangle IKT \implies \angle GHU = \angle KIT$

Gọi $A'$ là giao của $AM$ với $EF$. Ta có $\angle KDP = \angle KDH + \angle HDP = \angle KMH + \angle MHA' = \angle MA'Y$ nên $PDKA'$ nội tiếp

Ta có biến đổi góc $\angle  SDK = 180 -\angle SDG - \angle KDC = 180 - \angle SA'M - \angle GAA' = 180 - \angle AGA' - 2\angle GAA' $ và $\angle TDA' = 180 -\angle TDG - \angle A'DM = 180 - \angle GAM - \angle GAA'$. Suy ra $\angle TDA' = \angle SDK$, tức là $\angle TDS = \angle A'DK = \angle KPE = \angle GHU$. Ta có điều cần chứng minh

Untitled.png




#682404 Tuần 5 tháng 5/2017: Chứng minh rằng bốn điểm $R,H,J,K$ cùng thuộc...

Đã gửi bởi manhtuan00 on 30-05-2017 - 12:27 trong Chuyên mục Mỗi tuần một bài toán Hình học

lời giải sai 




#681584 Tuần 4 tháng 5/2017: Chứng minh rằng $MY \parallel KR$.

Đã gửi bởi manhtuan00 on 23-05-2017 - 01:59 trong Chuyên mục Mỗi tuần một bài toán Hình học

Lời giải bài 2 của em cũng dùng nghịch đảo nhưng đoạn sau hơi khác tẹo ạ

 $PH$ cắt $BC$ tại $L$ .Xét phép nghịch đảo đối xứng $I^A_{AB.AC}.R_\triangle$ : $P \leftrightarrow H \implies X \leftrightarrow L$. Từ đây suy ra $AL,AX$ đẳng giác

$AO$ cắt $BC$ tại $R$, $A'$ đối xứng $A$ qua $O$, $M$ là trung điểm $BC$. Theo Menelaus cho $\triangle HA'P$ cát tuyến $BC$ ta có $\frac{LH}{LP} = \frac{RA'}{RP} $. Mà $RA'.RA = RO.RP = RB.RC$ nên $\frac{RA'}{RP} = \frac{RO}{RA}$. Ta lại có $OP.OR = OA^2$ nên $\frac{OA}{OR} = \frac{OP}{OA} = \frac{AP}{AR}$. Kết hợp với các điều trên suy ra $\frac{AO}{AP} =\frac{LH}{LP}$. Lại theo Menelaus cho $\triangle HOP$ với cát tuyến $AL$, ta suy ra $AL$ đi qua trung điểm $OH$, tức là tâm Euler, suy ra $AL$ đi qua điểm đối xứng của $O$ qua $BC$. Mà $AL,AX$ đẳng giác nên $AX$ cũng đi qua điểm cố định trên trung trực $BC$

Untitled.png

 




#681583 Tuần 4 tháng 5/2017: Chứng minh rằng $MY \parallel KR$.

Đã gửi bởi manhtuan00 on 23-05-2017 - 01:22 trong Chuyên mục Mỗi tuần một bài toán Hình học

Lời giải bài 1 của em ạ :

Gọi $A_1$ đối xứng $A$  qua $K$. $Z,H$ lần lượt là chân đường đối trung và chân đường cao tại $A$.  Do $C,B$ đối xứng nhau qua $I$ nên $CA_1$ đi qua điểm đối xứng của $A$ qua trung trực $PQ$.  Gọi điểm này alf $S$. Ta có $ZA$ cũng là đường đối trung của $\triangle PZC$ nên $ZA,ZI$ đẳng giác trong $\angle PZC$. Vậy $ZI$ đi qua $S$. 

$AM$ cắt $KR$ tại $N$ . Ta cần chứng minh $\frac{AM}{AN} = \frac{AY}{AX}$ . Điều cần chứng minh tương đương với $\frac{AC}{AN} = \frac{EQ}{AP} (1)$

Ta sẽ chứng minh $AR$ là phân giác $angle MAK$. Thật vậy , ta có $\angle MAR = \angle MAP + \angle PAR  = \angle QAC + \angle IAQ = \angle IAC$. Lại có $\triangle AIZ \sim \triangle ACA_1$ nên $\angle ZAI = \angle CAA_1$ hay $\angle KAR = \angle IAC$. Vậy $AR$ là phân giác $\angle MAK$. 

Ta lại có $\angle KZA = \angle KAR = \angle RRAN$ nên $AN \parallel KZ$. Áp dụng Thales ta có $\frac{AN}{PZ} = \frac{RA}{RZ} = \frac{PA^2}{PZ^2} = \frac{IA^2}{IQ^2}$. Vậy $AN = R.\frac{IA^2}{IQ^2}$. $(2)$.

Gọi $T$ là trung điểm $QE$. Ta có $\angle TIQ = \angle CIQ - \angle CIT = \angle HIB - \angle CAT = \angle IAC - \angle TAC = \angle IAT$. Từ đây ta có $QI^2 = QT.QA = \frac{QE.QA}{2}$ hay $QE = \frac{2QI^2}{QA}$. $(3)$. Thế $(2), (3)$ vào $(1)$ điều cần chứng minh tương đương với : $\frac{AC}{R}.\frac{IQ^2}{IA^2} = \frac{2QI^2}{AQ.AP}$ hay $\frac{AC}{R.AI^2} = \frac{2}{AP.AQ}$

Thật vậy , $\frac{AI^2}{AH} = AC = AB$ nên ta cần chứng minh $\frac{AI^2}{R.AI^2.AH} = \frac{2}{AQ.AP}$ hay $\frac{1}{AH.R} = \frac{2}{AP.AQ}$. Điều này đúng do $\triangle HAP \sim \triangle QAA_1$

Ta hoàn tất chứng minh 

unnamed.png
 




#680943 Tuần 3 tháng 5/2017: đường thẳng $AQ$ luôn đi qua một điểm cố định...

Đã gửi bởi manhtuan00 on 16-05-2017 - 22:37 trong Chuyên mục Mỗi tuần một bài toán Hình học

Lời giải bài 1 : Gọi $G$ là tiếp điểm của $(K)$ với $(O)$. $X,Y,T$ lần lượt là giao của $AQ$ với $BC$, trung điểm cung lớn $BC$,  giao của tiếp tuyến tại $P$ với $BC$

Ta có $TP^2 = TB.TC$ nên $T$ nằm trên tiếp tuyến chung của $(K)$ và $(O)$. 

Áp dụng định lý Protassov cho $\triangle ABC$, đường tròn $(O)$ và đường tròn $(K)$, ta có $GI$ đi qua $Y$ do là phân giác . $GI$ cắt $BC$ tại $Z$

Áp dụng định lý Protassov cho $\triangle ABC$, đường tròn $(BPC)$ và đường tròn $(K)$, ta có $PJ$ là phân giác $\angle BPC$. Thật vậy, ta có $\frac{PB^2}{PC^2} = \frac{TB}{TC} = \frac{GB^2}{GC^2} = \frac{ZB^2}{ZC^2}$ nên $PZ$ là phân giác $\angle BPC$. Từ đây suy ra $P,Z,J$ thẳng hàng

Ta sẽ chứng minh $AQ$ đi qua $Y$. Thật vậy, ta giả sử $AY$ cắy $YP$ tại $Q$ thì ta cần chứng minh $MQ$ chia đôi $AH$. Gọi $L$ là giao điểm của $AJ$ với $BC$

Thật vậy ta cps : $M(AH,QY) = (AX,QY) =  Z(AX,QY) = Z(AL,JI) = (AL,JI) = -1$. Suy ra $MQ$ chia đôi $AH$. Vậy ta có điều cần chứng minh 

Untitled.png




#680090 Tuần 2 tháng 5/2017: Chứng minh rằng trung trực $HL$ chia đôi...

Đã gửi bởi manhtuan00 on 09-05-2017 - 18:11 trong Chuyên mục Mỗi tuần một bài toán Hình học

Lời giải bài 2 :  Gọi $M,N,R$ là giao điểm của $AQ,BQ,CQ$ với $(O)$. Theo Pascal đảo ta có $MX,NY,RZ$ đồng quy tại $U$

Khi đó ta có $(UZ,UC) = (AR,AC) = (AR,AB)+(AB,AC) = (CQ,CB)+ (AB,AC) = (CA,CF) + (AB,AC) = (FA,FC) = (FZ,FC) \implies $ tứ giác $YFUC$ nội tiếp. Vậy $U \in (CZF)$

Tương tự ta có $U \in (BYE) , U \in (AXD)$. Suy ra 3 đường tròn trên cùng với $(O)$ đồng quy tại $U$

18423716_1891690341044106_964129297620655782_n.jpg




#679955 Tuần 2 tháng 5/2017: Chứng minh rằng trung trực $HL$ chia đôi...

Đã gửi bởi manhtuan00 on 08-05-2017 - 13:22 trong Chuyên mục Mỗi tuần một bài toán Hình học

Lời giải bài 1 : 

Gọi $A'$ đối xứng $A$ qua $O$. $EF$ cắt $BC$ tại $X$. $A'X$ cắt $(O)$ tại $K$
Khi đó $A,J,K,X,D$ đồng viên vì cùng nằm trên đường tròn $(AX)$.
Gọi $V$ là giao của $(AH)$ với $(O)$. Khi đó $\overline{A,V,X} $ và $ \overline{V,H,M,A'}$
Ta cần chứng minh $MH = ML $, tức là $L$ đối xứng $A'$ qua trung trực $EF$
Gọi $G$ là giao điểm của $AM$ với $(AEF)$. Ta có do $AM,AR$ đẳng giác trong $\angle BAC$ nên $G,R$ đối xứng nhau qua trung trực $EF$
Xử dụng phép đối xứng trục qua trung trực $EF$, ta đưa bài toán về chứng minh $A',P'G$ thẳng hàng. Thật vậy, nếu ta giả sử $P$ là giao điểm của $A'G$ với $KH$, gọi $W$ là giao điểm của $GK$ với $HA'$. Ta có : $X(PW,HK_)= -1$. Ta cần chứng minh $X(EW,HK) = -1$
Gọi $T$ là giao điểm của $HV$ với $EF$. Xét phép nghịch đảo $I^A_{AB.AF = AC.AE}$ biến $V \rightarrow X, H \rightarrow D, EF \rightarrow (O) \implies T \rightarrow K$
Suy ra $\overline{A,T,K}$. Gọi $Z$ là giao điểm của $AM$ với $EF$ ; $U$ là giao điểm của $GT$ với $XK$
Vậy ta có biến đổi tỉ số $X(EW,HK) = X(TW,HA') = G(TW,HA') = G(UK,XA') = T(UK,XA') = T(GA,ZM) = X(GA,ZM) = X(HA,FB) = -1$

Vậy điều cần chứng minh là đúng nên $A',G,P$ thẳng hàng, suy ra $M$ nằm trên trung trực $LH$

Untitled.png




#679828 Đề thi Olympic chuyên KHTN 2017

Đã gửi bởi manhtuan00 on 07-05-2017 - 13:38 trong Thi HSG cấp Tỉnh, Thành phố. Olympic 30-4. Đề thi và kiểm tra đội tuyển các cấp.

Em xin phép giải câu hình 2 ạ 

a, Gọi $U$ là giao điểm của $KR$ với $BC$

Ta có $KT.KF = KB^2 $ nên $\angle KTB = \angle KBF = \angle KAF$. Suy ra $ADFT$ nội tiếp. Khi đó ta có $LD.LT = LF.LA = LB .LC = LP.LQ$ nên tứ giác $TPDQ$ nội tiếp

Lại có $KR.KU = KC^2$ nên tứ giác $ADRU$ nội tiếp. Và $KQ^2 = KC^2 = KD.KA$ nên ta có $\angle DUR = \angle DAR = \angle KQD = \angle PTD$. Từ đây suy ra $KR \parallel TP$

b,Gọi $M$ là giap điểm của $PK$ với $(O)$.Ta có $KL.KM = KB^2 = KP^2 = KQ^2$ nên $(ML,PQ) = -1 = A(ML,PQ) = A(MF,ER) = (MF,ER) = K(MF,ER) \implies KE$ chia đôi $TP$

Untitled.png

 




#679826 Đề thi Olympic chuyên KHTN 2017

Đã gửi bởi manhtuan00 on 07-05-2017 - 12:58 trong Thi HSG cấp Tỉnh, Thành phố. Olympic 30-4. Đề thi và kiểm tra đội tuyển các cấp.

Lời giải bài đa thức

Thế $a = x+1,b = -2x-1,c = -2x-3$, khi đó ta có $P^2(x+1)+P^2(-2x-1)+P^2(-2x-3) = P^2(-3x-3)+2$

So sánh hệ số bậc $2n$ ta có $1+(-2)^n+(-2)^n = (-3)^n$.

Tức là $(-1)^{n+1} = 3^n-2^{n+1}$. Điều này chỉ đúng khi $ n =1$

Vậy $P(x) = ax+b$. Thay vào ta có $a2+b^2 = 1$




#679617 Tuần 1 tháng 5/2017: Chứng minh rằng $JH \perp IO$.

Đã gửi bởi manhtuan00 on 05-05-2017 - 21:15 trong Chuyên mục Mỗi tuần một bài toán Hình học

Lời giải bài 2 của em ạ

Gọi $X_a,X_b,X_c$ là các đỉnh tam giác đều dựng ra bên ngoài $\triangle ABC$. $Y_a,Y_b,Y_c$ là các đỉnh tam giác đều dựng ra bên trong $\triangle ABC$

Gọi $U_a,V_a$ là các đỉnh tam giác đều dựng vào phía trong $\triangle F_1BC$. Tương tự ta có $U_b,U_c,V_b,V_c$

Do $F_a$ là điểm Fermat thứ hai của $\triangle F_1BC$ nên $F_1BF_aU_a, F_1CV_aF_a$ nội tiếp

Chứng minh tương tự ta có các tứ giác nội tiếp $F_bV_bCF_1,AF_bU_bF_1,AV_cF_cF_1,F_1U_cF_cB$

Do $\angle AF_1V_b = \angle V_bCF_1 = 60^{\circ}$ nên $AF_1$ tiếp xúc $F_bV_bCF_1$. Từ đây suy ra $AF_b.AV_b = AF_1^2$

Tương tự ta có $AF_1^2 = AU_c.AF_c$ nên tứ giác $F_cU_cF_bV_b$ nội tiếp. Tương tự ta cũng có các tứ giác nội tiếp $F_bU_bU_aF_a,V_aF_aF_cV_c$

Ta có $\angle BF_2C = \angle BY_aC = BU_cC = \angle BV_bC = 180^{\circ} - \angle BF_aC = 60^{\circ}$ nên các điểm $U_c,F_2,Y_a,V_b,C,F_a,B$ đồng viên

Tương tự ta có $A,F_b,C,V_a,Y_b,V_c,F_2$ đồng viên

Từ đây ta có biến đổi góc $\angle F_2F_bF_c = \angle AF_bF_c - \angle AF_bF_2 = \angle AU_cV_b - \angle AF_bF_2 $ 

Và $\angle F_2F_aF_c = \angle F_2F_aB - \angle F_cF_aB = \angle F_2F_aB - \angle BV_cV_a= \angle F_2CB - \angle BV_cV_a$

Ta cần chứng minh $\angle F_2F_aF_c = \angle F_2F_bF_c$. Điều này tương đương với $\angle AU_cV_b+ \angle BV_cV_a = \angle ACB $

18058037_1300066020029393_3408771786302823466_n.jpg

Khi và chỉ khi $\angle AU_cF_1 +\angle BV_cF_1 = \angle F_1BC+\angle F_1AC+\angle BAC = 120^{\circ}$

Điều này tương đương với $\angle U_cBV_c = \angle U_cAV_c$

Thật vậy ta có do $BF_cAF_1$ là hình bình hành nên $F_cA = BF_1 = BV_c$

Mà $V_cU_c = V_cF_c$ suy ra $\triangle U_cV_cB = \triangle V_cF_cA \implies \angle U_cBV_c = \angle U_cAV_c$. Vậy ta có điều cần chứng minh là đúng

Vậy $F_a,F_b,F_c,F_2$ đồng viên 

13r.png




#679550 Tuần 1 tháng 5/2017: Chứng minh rằng $JH \perp IO$.

Đã gửi bởi manhtuan00 on 05-05-2017 - 01:07 trong Chuyên mục Mỗi tuần một bài toán Hình học

Lời giải bài 1 : 

Gọi $BX,CY$ là các đường phân giác của $(O)$ với $X,Y \in (O)$. Gọi $R$ là giao điểm của $IK$ với $XY$. Theo định lý con bướm thì $I$ là trung điểm $KR$ nên $R$ là đối xứng của $K$ qua $OI$. Mà $M,N$ đối xứng nhau qua $OI$ nên $\angle RMI$ đối xứng $\angle KNI$ qua $OI$, tức là $\angle KNI = 90^{\circ}$

Gọi $Q$ là giap điểm của $KN$ với phân giác ngoài $\angle BAC$, $S$ là trung điểm $AN$. Khi đó $H$ chính là đối xứng của $Q$ qua $S$.

Gọi $P$ đối xứng $J$ qua $S$. Ta cần chứng minh $QP \parallel OI$.

Do $MN \parallel OI$ nên $J$ là trung điểm $AK$. Vậy nếu gọi $R$ là giao điểm của $AP$ với $KN$ thì $P$ là trung điểm $AR$, $N$ là trung điểm $KR$.

Ta có $\angle QAR = 90 - \angle IAR = 90 - \angle IMN = 90 - \angle INM = \angle MNQ = \angle ARQ$ nên $\triangle QAR$ can. Mà $P$ là trung điểm nên $QP \perp AR$

Lại có $AR \perp  OI$ nên $QP \parallel OI$. Suy ra $JH \parallel OI$

13r.png




#678241 Tuần 3 tháng 4/2017: Chứng minh rằng đường thẳng $QR$ đi qua điểm c...

Đã gửi bởi manhtuan00 on 21-04-2017 - 20:18 trong Chuyên mục Mỗi tuần một bài toán Hình học

Lời giải bài 1 : Ta sẽ chứng minh $QR$ đi qua $O$. Ta sẽ chứng minh $AL,OQ,BC$ đồng quy

Gọi $X,Y,Z$ tam giác Pedal của $Q$ lên $\triangle ABC$.G ọi $N$ là giao điểm của $QX$ với $(XYZ)$, ta có : $\angle NQZ = \angle ZBX = \angle DPF$

Mặt khác ta có $\angle DFP = \angle DBP = \angle CQP = 180^{\circ} - \angle AQC = \angle QAY + \angle QCY = \angle QZY + \angle QXY = \angle QZY + \angle NZY = \angle NZQ$

Từ hai điều trên ta có $\triangle NZQ \sim \triangle DFP$. Vậy phép vị tự tâm $A$ biến $Y$ thành $E$, biến $Z$ thành $F$, biến $Q$ thành $P$ sẽ biến $N$ thành $D$, biến $X$ thành $K$.

$DP$ cắt $OQ$ tại $G$, $I,J$ là trung điểm cung nhỏ, cung lớn $\overarc {BC}$, $J$ là hình chiếu của $K$ lên $EF$, $KD$ cắt $EF$ tại $R$.

Gọi $B',C'$ là các điểm nằm trên phân giác ngoài sao cho $XB' \perp CQ$ và $BC' \perp BQ$ và $M$ là trung điểm $YZ$
Khi đó $V$ là trung điểm $B'C'$ . Lại có $\angle QB'A = \angle QCA = \angle NZY$. Vậy $\triangle NYZ \cap M \sim \triangle QC'B' \cap V \implies QV \parallel NM$
Gọi $A'$ là chân đường phân giác trong . Ta có $A'B.A'C = A'I.A'A = A'P.A'Q =\implies \frac{A'I}{IP} = \frac{A'P}{PA}$.Lại có $\triangle VIB \sim \triangle AQZ$ nên $\frac{QM}{QA} = \frac{IB^2}{IV^2}$
Mà $\triangle QA'X \sim \triangle IVA \implies \frac{QA'}{QX}= \frac{IV}{IA}$ nên từ đây ta có $\frac{QM}{QA} .\frac{QA'}{QX} = \frac{IB^2}{IV.IA} = \frac{IA'.IA}{IV.IA} = \frac{IA'}{IV} \implies \frac{QM}{QX} = \frac{A'T}{A'Q}.\frac{QA}{IV} = \frac{IP}{IV}$. Suy ra $\triangle MQX \sim \triangle PIV$ nên $VP \parallel MX$
Gọi $H,T$ là giao điểm của $PD$ với phân giác ngoài và $VQ$. Khi đó $\frac{KD}{KR} = \frac{XN}{XT} = \frac{PT}{PH} = \frac{AI}{AP}.\frac{QP}{QI}$

Ta có $\frac{PG}{PA} = \frac{OI}{PA}.\frac{QP}{QI} = \frac{IV}{2IA}.\frac{IA}{PA}.\frac{QP}{QI} =  \frac{1}{2}\frac{KR}{KJ}.\frac{KD}{KR}= \frac{KD}{KL}$. Suy ra $AG \parallel LD$

Đường thẳng qua $D$ song song $AP$ cắt $AL$ tại $W$. Ta chứng minh $GW \parallel UQ$. 

Lấy điểm $V$ trên tia đối của $AP$ thỏa mãn $\frac{AV}{AP} = \frac{AW}{AL}$. Khi đó $VW \parallel LP$. Ta cần chứng minh $V,W,G$ thẳng hàng. Điều này tương đương với $\frac{PV}{PG} = \frac{KL}{PL}$, hay $\frac{PA}{PG} . \frac{KD}{KP} = \frac{KL}{KD}.\frac{KD}{KP}$, hay $\frac{PA}{PG} = \frac{KL}{KD}$. Điều này đúng do $\triangle AGP$ và $\triangle LDK$ có các cạnh tương đối song song. Vậy $GW \parallel PL \parallel UQ$

Xét $\triangle WGD$ và $\triangle UQX$ có các cạnh tương đối song song nên $WU,QG,XD$ đồng quy, tức là $AL,OQ,BC$ đồng quy. Ta hoàn tất chứng minh

 

 

123.png




#677103 $\textbf{Đề thi MYTS vòng 2 Lớp 9}$

Đã gửi bởi manhtuan00 on 11-04-2017 - 19:14 trong Tài nguyên Olympic toán

câu 4 chia thành 7 hình tháp 3 ô, mỗi hình tháp tổng chẵn nên có 1 số chẵn suy ra đs là 7




#677099 Tuần 2 tháng 4/2017: Chứng minh rằng $\frac{MP}{NQ...

Đã gửi bởi manhtuan00 on 11-04-2017 - 18:40 trong Chuyên mục Mỗi tuần một bài toán Hình học

Lời giải bài 2 : Gọi $S$ là giao điểm của $DX$ với $(O)$. $AQ$ cắt $(O)$ tại $L$. Khi đó $LS$ là đường kính của $(O)$

Gọi $X'$ là đối xứng của $X$ qua $O$ khi đó $LX \perp BC$. Gọi $M$ là hình chiếu của $Q$ lên $BC$. $X'M$ cắt $BC$ tại $I$

$\blacksquare$ Ta chứng minh $U,V,M,D$ đồng viên

Ta sẽ chứng minh $XI,BC,EF$ đồng quy. Gọi $EF$ cắt $BC$ tại $G_1$ thì $\frac{G_1B}{G_1C} = \frac{FB}{FA}.\frac{EA}{EC}$

Gọi $XI$ cắt $BC$ tại $G_2$ thì $\frac{G_2B}{G_2C} =  \frac{IB}{IC}.\frac{XB}{XC}$

Vậy điều cần chứng minh tương đương với 

$\frac{FB}{FA}.\frac{EA}{EC} = \frac{IB}{IC}.\frac{XB}{XC}$

Thật vậy ta có : $\frac{MB}{MC} = \frac{IB}{IC}.\frac{X'B}{X'C} = \frac{IB}{IC}.\frac{AF}{AE} = \frac{IB}{IC}.\frac{AF}{AE}$ nên $\frac{IB}{IC} = \frac{MB}{MC}.\frac{AE}{AF}$ ( do có $\triangle X'BC \sim \triangle AFE$)

Vậy ta cần chứng minh $\frac{AE}{AF}.\frac{BF}{CE}= \frac{MB}{MC}.\frac{AE}{AF}.\frac{XB}{XC}$ tức là $\frac{MB}{MC} = \frac{FB}{EC}.\frac{XB}{XC}  $

Có $\triangle FPE \sim \triangle BXC$ nên $ \frac{FB}{EC}.\frac{XB}{XC}  =  \frac{FB}{EC}.\frac{PF}{PE} = \frac{cotan \angle FBP}{cotan \angle ECP} = \frac{cotan \angle  QBM}{cotan \angle QCM} = \frac{MB}{MC}$. Ta có điều cần chứng minh

Vậy $XI,BC,EF$ đồng quy tại $G$

$\blacksquare$ Ta chứng minh đối xứng của $Q$ qua $K$ nằm trên $(O)$

Thật vậy, ta sẽ chứng minh $S$ chính là đối xứng của $Q$ qua $K$

Gọi $K'$ là trung điểm $QS$. Do $S$ là đối xứng của $L$ qua $O$ nên $AS \perp AQ$ , từ đây suy ra $AS \parallel UV$

$K'$ là trung điểm $QS$ nên $K'$ nằm trên trung trực $AS$, cũng là trung trực $UV$

Lại có $QMDS$ là hình thang vuông nên $K'$ cũng nằm trên trung trực $MD$. Mà tứ giác $MDVU$ nội tiếp nên $K'$ chính là tâm ngoại tiếp $MDVU$ nên $K \equiv K'$

Vậy $K$ chính là trung điểm $QS$.

Hình gửi kèm

  • DSC_0121.JPG



#677092 Tuần 2 tháng 4/2017: Chứng minh rằng $\frac{MP}{NQ...

Đã gửi bởi manhtuan00 on 11-04-2017 - 17:01 trong Chuyên mục Mỗi tuần một bài toán Hình học

Lời giải bài 1 : 

Gọi $X$ là giao điểm của $CK$ với $(O)$ , $OK$ cắt $AX$ tại $S$. Khi đó $\angle XKS  = \angle OCA = 90^{\circ} - \angle AXK$ nên $OS \perp XK$

Tức là $KA = KX$ nên $X \in (K,KA)$. 

Ta có : $\angle PQC = 180^{\circ} - \angle QND - \angle QDN = 180^{\circ} - \angle ANM - \angle AXC = (90^{\circ} - \angle XNM) +90^{\circ} - \angle ANX - \angle ABC = (90^{\circ} -\angle XCB) +90^{\circ} - \angle OCA - \angle ABC = 90^{\circ} - \angle XCB$ nên $CX$ đi qua $J$, tức là $L,J,K,X$ thẳng hàng

$(K)$ cắt $XC$ tại $G$. Khi đó $\angle MGJ = \angle MAX = \angle BCG$ nên $GM \parallel BC$. Tương tự ta có $GN \parallel DC$ nên $\frac{JM}{JP} = \frac{JG}{JC}=\frac{JN}{JQ}$ nên $\frac{MP}{NQ}=\frac{JM}{JN}$

Hình gửi kèm

  • DSC_0121.JPG



#676760 Đề thi $Olympic$ $30/4$ lớp $11$ năm $2017...

Đã gửi bởi manhtuan00 on 09-04-2017 - 17:17 trong Thi HSG cấp Tỉnh, Thành phố. Olympic 30-4. Đề thi và kiểm tra đội tuyển các cấp.

Bài 5 :  Áp dụng bổ đề quen thuộc sau : $S(n(10^k-1)) = 9k$ với mọi $n <10^k-1$

Thật vậy, ta có nếu $ 3|n$ thì $S(xn) \equiv xn \equiv 0 $ (mod $3$) nên $S(xn)$ không thể nhận mọi số dư modulo $n$

Nếu $(n,3) = 1$ ; Ta chọn $x = 10^t -1$ với $t > log_{10}(n+1)$ và $9t \equiv k$ (mod $n$). Khi đó $S(n(10^t-1)) = 9t \equiv k $ (mod $n$)

Vậy đáp số là tất cả những số không chia hết cho $3$